You are on page 1of 21

MỘT SỐ BÀI TOÁN

LIÊN QUAN ĐẾN CHỦ ĐỀ ĐA THỨC1


Nguyễn Duy Thái Sơn

(Đại học Sư phạm – Đại học Đà Nẵng)

Qua bài giảng này, ta sẽ khảo sát một số dạng toán về đa thức, thường gặp trong các kỳ thi chọn học
sinh giỏi THPT quốc gia hoặc trong các kỳ thi chọn đội tuyển thi Toán quốc tế. Ta cũng sẽ gặp một
số dạng toán Olympic mà phát biểu của chúng có vẻ ngoài không liên quan gì đến đa thức, nhưng
các kỹ thuật đa thức lại cho ta những lời giải đẹp. Và – vẫn như ở các bài giảng trước đây của tôi –
điều quan trọng hơn cả mà tôi muốn truyền tải không chỉ là một lời giải (sẽ được ghi ra), đó phải là
các phân tích, định hướng trước khi ta đi đến được lời giải của một bài toán…

Bài toán 1. Cho bốn số thực a, b, c, d đôi một phân biệt.

 x  ay  a 2 z  a 3t  a 4

 x  by  b z  b t  b
2 3 4

(i) Giải hệ phương trình: 


 x  cy  c z  c t  c
2 3 4

 x  dy  d 2 z  d 3t  d 4 .

 x y z t
 a 1  a  2  a  3  a  4  1

 x  y  z  t 1
b 1 b  2 b  3 b  4
(ii) Giả sử {a; b; c; d }  {1; 2;3; 4}  . Giải hệ phương trình: 
 x  y  z  t 1
c 1 c  2 c  3 c  4
 x y z t
     1.
 d 1 d  2 d  3 d  4

1
Đây là nội dung bài giảng được tác giả trình bày tại các lớp Tập huấn Giáo viên THPT chuyên Khu vực miền Bắc (Bắc Ninh,
16-23/7), Khu vực miền Trung (Nghệ An, 23-30/7) và Khu vực miền Nam (Bình Dương, 30/7-6/8) do ``Chương trình trọng điểm
Quốc gia phát triển Toán học giai đoạn 2010-2020'', Viện Nghiên cứu cao cấp về Toán, tổ chức năm 2017.
Lời giải. (i) Với mỗi bộ ( x; y; z; t )   4 , đặt P ( s ) : s 4  ts 3  zs 2  ys  x (s ), ta thu được một đa
thức P  [s ] có deg P  4 và có hệ số của s 4 (hệ số bậc cao nhất) là 1. Do a, b, c, d đôi một phân
biệt, dễ thấy: ( x; y; z; t ) là một nghiệm của hệ đã cho khi và chỉ khi {a; b; c; d } là tập nghiệm của
phương trình P ( s )  0 (với s là ẩn số). Theo định lý nhân tử hóa đa thức, điều này tương đương
với:

P ( s )  ( s  a )( s  b)( s  c)( s  d )
   
 s 4  ts 3  zs 2  ys  x  s 4    a  s 3  (ab  ac  ad  bc  bd  cd ) s 2    abc  s  abcd
 cyc   cyc 
 x   abcd

 y   abc
 cyc

 z  (ab  ac  ad  bc  bd  cd )
t  a.
  cyc

(ii) Với mỗi bộ ( x; y; z; t )   4 , đặt

P ( s ) : ( s  1)( s  2)( s  3)( s  4)  x( s  2)( s  3)( s  4)  y ( s  1)( s  3)( s  4)  z ( s  1)( s  2)( s  4)


 t ( s  1)( s  2)( s  3),

ta thu được một đa thức P  [s ] có deg P  4 và có hệ số bậc cao nhất là 1. Dễ thấy: ( x; y; z; t ) là
một nghiệm của hệ đã cho khi và chỉ khi {a; b; c; d } là tập nghiệm của phương trình P ( s )  0; một
cách tương đương,

P ( s )  Q( s ); (1)

trong đó, Q ( s ) : ( s  a )( s  b)( s  c)( s  d ). Ta cũng thấy: Q  [s ] có deg Q  4 và có hệ số bậc


cao nhất là 1. Suy ra: deg( P  Q)  4. Vậy, P  Q là đa thức không khi và chỉ khi P  Q triệt tiêu
tại 4 điểm đôi một phân biệt:

(1)  P (1)  Q (1)  P (2)  Q (2)  P (3)  Q (3)  P (4)  Q (4)  0. (2)

Nhưng

P (1)   x(1  2)(1  3)(1  4)  6 x, P (2)   y (2  1)(2  3)(2  4)  2 y,


P (3)   z (3  1)(3  2)(3  4)  2 z , P (4)  t (4  1)(4  2)(4  3)  6t ,

nên
(2)  x  Q(1) 6, y   Q (2) 2, z  Q(3) 2, t   Q (4) 6
 x  (a  1)(b  1)(c  1)(d  1) 6
 y   (a  2)(b  2)(c  2)( d  2) 2


 z  (a  3)(b  3)(c  3)(d  3) 2
t   (a  4)(b  4)(c  4)( d  4) 6.

Bài toán 2. Cho 2016 số thực c1 , c2 ,  , c2016 thỏa hệ:

 c1 c2 c2016 1
    
23 3 4 2017  2018 1  3

 c1 c
 2
 
c2016

1
 3 4 45 2018  2019 3  5
   

 c1

c2
 
c2016

1

 2017  20182018  2019 4032  4033 4031  4033
c c c2016
Hãy tìm giá trị của tổng: S  1  2    
1 2 2  3 2016  2017
Lời giải. Xét đa thức Q  [x] (deg Q  2017) và phân thức f  ( x) được cho bởi:

2017 2016
ci
Q ( x) :  ( x  i ), f ( x) :  
i 1 i 1 ( x  i )( x  i  1)

P( x)
Dễ thấy: tồn tại đa thức P  [x] có deg P  2015 để f ( x)   Theo hệ phương trình trong đề
Q( x)
bài, với mỗi số nguyên j mà 1  j  2016, ta đều có:
1
f ( j)   (4 j 2  1) P ( j )  Q( j )  0.
(2 j  1)(2 j  1)
Đặt R ( x) : (4 x  1) P ( x)  Q( x), ta thu được đa thức R  [x] có deg R  2017 và thỏa: R ( j )  0
2

với mọi số nguyên j mà 1  j  2016. Vậy, theo định lý nhân tử hóa đa thức, tồn tại các hằng số
thực  ,  sao cho
2016 2016
R ( x)  ( x   ) ( x  j )  (4 x 2  1) P( x)  Q( x)  ( x   ) ( x  j ). (1)
j 1 j 1

Trong (1), chọn x : 1 2 ta suy ra


2017
1  1  1 
  2  i    2016   2017 
   2 
1 Q(1 2) 2
     2016   2016i 1
(1) 2016
1   1  1
 
2
  j   j
j 1  2  j 1  2  2
   2   4033  4035. (2)
Còn với x : 1 2 thì (1) kéo theo:
2017
 1 
1 Q(1 2)    2  i  1
      2016  i 1
(1) 2016  
 1  2016
1 
   2  j 
2 2
  j
j 1  2  j 1

   2   1. (3)
  1  2  2017
2

Giải (2)-(3) ta tìm được 


   2017 .
2

Cuối cùng, trong (1) chọn x : 0, ta có :


2016
 P(0)  Q(0)    ( j )  2017 2  2016! ( 1) 2016  2016! 2017 2
j 1

P (0) 2016! 2017 2 2016! 2017 2


 f (0)   1   1 
Q(0) Q(0) 2017!
2016
ci
S  f (0)  1  2017  2016.
i 1 i (i  1)
m
Bài toán 3 (Romania). Với mỗi cặp số nguyên m, n mà 1  m  n, đặt Rnm :  (m  k ) n (1) k C kn 1.
k 0

Với các số nguyên m, n như thế, chứng minh rằng Rnn m1  Rnm .

Lời giải. Trước tiên, để ý: 1  m  n  1  n  m  1  n. Từ đó, theo định nghĩa,


n  m 1 n  m 1
Rnn  m1   (n  m  1  k ) n (1) k C kn 1    (n  1  k )  m  (1) k C nn11 k
n

k 0 k 0
n 1
  (i  m) n (1) n 1i Cin 1 (thay i : n  1  k  k  n  1  i )
i m
n 1
  (m  i ) n (1) n (1) n 1i Cin 1
i m
n 1
   (m  k ) n (1) k C kn 1 (thay k : i ).
k m

So sánh với công thức xác định của Rnm , và chú ý rằng (m  k ) n (1) k C kn 1  0 khi k  m, ta thấy
Rnn m1  Rnm khi và chỉ khi

n 1

 (m  k ) (1) C
k 0
n k k
n 1  0. (1)
Để chứng minh (1), ta sẽ sử dụng công thức nội suy Lagrange: Cho P  [x], deg P  n, và cho
n  1 số thực x1 , x2 , , xn 1 đôi một phân biệt. Với mọi số nguyên 1  k  n  1 và với mọi số thực x,
đặt

 (x  x ) i
 ( x) : ik

 (x  x )
k
k i
ik

(trong các tích ở trên, i chạy từ 1 đến n  1 và khác k). Khi đó ta có đồng nhất thức
n 1
P ( x)   P ( xk )k ( x). (2)
k 1

Chứng minh: Với mọi số nguyên 1  k  n  1, 1  j  n  1, dễ thấy

1 khi j  k
k ( x j )  
0 khi j  k .

Từ đó, hai vế của (2) bằng nhau tại n  1 điểm x1 , x2 , , xn 1. Cả hai vế đều là các đa thức (của biến
số x) có bậc không vượt quá n, nên chúng phải trùng nhau.

Trở lại bài toán, xét đa thức P ( x) : (m  x) n và các điểm x j : j (1  j  n  1). Sử dụng (2) ta có:

n 1
P (0)   P (k )k (0)  0. (3)
k 1

Nhưng

 ( i )  k (n  1)! (n  1)!(1) n
 (0)  ik
 (1)n
  (1) k C nk 1
 (k  i)  k  (k  i)   (k  i)  k (n  1  k )!(1) n  k 1
 k!
k

ik i k i k

nên (3)  (1).

Nhận xét. Qua lời giải trên, thực chất ta đã chứng minh được đẳng thức
n 1

 (1) C
k 0
k k
n 1 P(k )  0

với mọi P  [x] mà deg P  n.

Bài toán 4 (P48, Tạp chí Pi). Tìm tất cả các số nguyên n  2 và tất cả các bộ n số nguyên
a1  a2    an

sao cho đa thức

S ( x)  ( x  a1 )  ( x  a2 )    ( x  an )  3

phân tích được thành tích của hai đa thức khác hằng, với hệ số nguyên.

Lời giải. Giả sử n  2 và a1  a2    an là các số nguyên sao cho ta có phân tích

n
S ( x)   ( x  ai )  3  G ( x)  H ( x); (1)
i 1

trong đó, G ( x ) và H ( x ) là hai đa thức khác hằng, với hệ số nguyên, mà không mất tính tổng quát,
ta cũng có thể giả sử deg G  deg H . Rõ ràng, hệ số bậc cao nhất của G ( x ) và H ( x ) phải cùng
bằng 1, hoặc cùng bằng 1. Nếu cần, sẽ đổi dấu cả G ( x ) và H ( x ), ta có thể xem hệ số đó bằng 1.

Từ nay về sau, để cho tiện, ta quy ước:

– mỗi số nguyên i , mà 1  i  n, sẽ được gọi tắt là một chỉ số;

– số phần tử của mỗi tập hợp hữu hạn X sẽ được ký hiệu bởi X .

Lần lượt xét các trường hợp:

1. Trường hợp n  4.
n
Lúc này, nếu deg H  1, tức là H ( x )  x  a với a   nào đó, thì theo (1),  (a  a )  3. Điều
i 1
i

này là vô lý, vì 3 không thể biểu diễn được thành tích của nhiều hơn ba số nguyên đôi một phân
biệt. Vì vậy,

n  2  deg G  deg H  2. (2)

Đặt R ( x ) : G ( x )  H ( x ). Ta có các nhận xét sau:

Nhận xét 1: R ( x ) là một đa thức khác hằng, với hệ số nguyên, và deg R  n  2.

Chứng minh: Giả sử phản chứng rằng R ( x)  c là một đa thức hằng. Khi đó, (1) được viết lại thành

 ( x  a )  3  cG ( x)   G ( x) 
2
i .
i 1
Trong đồng nhất thức trên, hệ số bậc cao nhất của đa thức ở vế trái là 1, của đa thức ở vế phải là
1; và ta gặp mâu thuẫn. Vậy, R( x) không thể là đa thức hằng. Các kết luận còn lại của Nhận xét 1
là hiển nhiên.

Nhận xét 2: R (ai )  {  2; 2} với mỗi chỉ số i.

Chứng minh: Do (1) ta thấy

G (ai )  H (ai )   3   G (ai ); H (ai )    3;1 ,  3; 1 , 1; 3 ,  1;3


 R (ai )  2

với mọi chỉ số i.

Nhận xét 3: Với mọi bộ ba chỉ số i, j , k mà i  j và R (ai )  R (a j )  R (ak ), ta có

( ak  ai )( ak  a j ) | 4.

Chứng minh: Đặt c  R (ak ). Theo Nhận xét 2, c  2;2 và R (ai )  R (a j )  c. Áp dụng định lý
Bezout (và Nhận xét 1), ta có phân tích R ( x)  c  ( x  ai )( x  a j ) P ( x), trong đó P ( x ) là một đa
thức với hệ số nguyên. Lấy x  ak ta suy ra

( ak  ai )( ak  a j ) | R ( ak )  c | 2 | c | 4.

Nhận xét 4: ( ai  a j ) | 4 với mọi cặp chỉ số i, j mà R (ai )  R (a j ).

Chứng minh: Đặt c  R (ai ). Theo Nhận xét 2, c  2;2 và R (a j )  c. Áp dụng định lý Bezout
(và Nhận xét 1), ta có phân tích R ( x)  c  ( x  a j ) P ( x), trong đó P ( x ) là một đa thức với hệ số
nguyên. Lấy x  ai ta suy ra

( ai  a j ) | R ( ai )  c | 2 | c | 4.

Dùng các nhận xét trên ta có thể loại đi hai trường hợp “con”: n  6, n  5.

1a/ Thật vậy, giả sử n  6. Vì a1  a2    an , với mọi chỉ số i  6, ta có ai  a1  4. Vậy, theo


Nhận xét 4, ta phải có

R (ai )  R (a1 ) với mọi chỉ số i  6. (3)

Đặc biệt, R (a6 )  R (a1 ). Mà ( a4  a6 )( a4  a1 )  2  3  4, nên theo Nhận xét 3 ta phải có


R (a6 )  R (a1 )  R(a4 ). Theo cách hoàn toàn tương tự, do ( a2  a6 )( a2  a4 )  4  2  4 và do
( a5  a1 )( a5  a2 )  4  3  4 nên Nhận xét 3 cho ta:
R (a1 )  R (a6 )  R (a4 )  R (a2 )  R (a5 ). (4)

Đặt c  R ( a1 ), từ (3) và (4) ta thấy phương trình R ( x)  c  0 có nhiều hơn n  2 nghiệm; đó là các
nghiệm x  ai với mỗi chỉ số i  3. Điều này mâu thuẫn với Nhận xét 1. Vậy, 1a/ không thể xảy ra.

Tiếp theo, gọi I là tập hợp tất cả các chỉ số i mà R (ai )  2, và J là tập hợp tất cả các chỉ số i mà
R (ai )  2. Theo Nhận xét 2, hiển nhiên ta có

I  J  n. (5)

1b/ Bây giờ giả sử n  5. Lúc này từ (1) và (2) dễ thấy deg G  3, deg H  2. Suy ra: R ( x)  2 và
R ( x)  2 đều là các đa thức bậc ba có hệ số bậc cao nhất bằng 1. Như là một hệ quả, ta có
I  3, J  3. Kết hợp với (5) ta thấy chỉ có hai khả năng:

(b1) I  3 và J  2

(b2) I  2 và J  3.

Xét (b1). Giả sử I  i1  i2  i3  . Theo định lý Bezout, ta có phân tích

R ( x)  2  ( x  ai1 )( x  ai2 )( x  ai3 ). (6)

Nếu 1  J , thì từ (6) suy ra 4  R (a1 )  2  (a1  ai1 )(a1  ai2 )(a1  ai3 ) ; nhưng điều này là không
thể, vì

1  a1  ai1  a1  ai2  a1  ai3  (a1  ai1 )(a1  ai2 )(a1  ai3 )  1 2  3  6.

Vậy, 1  I , tức là i1  1. Lập luận tương tự, ta có 5  I , tức là i3  5. Chỉ còn phải khảo sát ba giá trị
có thể của i2 .

(b1.1) Nếu i2  2, thì 4  J nên từ (6) suy ra

4  R(a4 )  2  ( a4  a1 )( a4  a2 )(a4  a5 )  3  2  1  6,

vô lý!

(b1.2) Nếu i2  3, thì 2  J nên từ (6) suy ra

4  R (a2 )  2  (a2  a1 )(a2  a3 )(a2  a5 )  0,

vô lý!
(b1.3) Nếu i2  4, thì 3  J nên từ (6) suy ra

4  R (a3 )  2  (a3  a1 )(a3  a4 )(a3  a5 )  0,

cũng vô lý!

Tất cả các mâu thuẫn gặp được cho thấy khả năng (b1) không thể xảy ra.

Ta có thể lặp lại phương pháp trên để thấy (b2) cũng không thể xảy ra. Một phương pháp khác là
quy (b2) về (b1) bằng cách thay dãy a1  a2    a5 bởi dãy

b1   a5  b2   a4    b5   a1 ,

và thay đa thức R( x) bởi đa thức T ( x)   R ( x). Chú ý rằng: T ( x)  2 và T ( x)  2 cũng là các đa


thức bậc ba với hệ số nguyên và có hệ số bậc cao nhất bằng 1; ngoài ra,

R (ai )  2  T (b6i )  2; R(ai )  2  T (b6i )  2

nên vai trò của các tập I , J đã được hoán đổi qua cách thay nói trên.

Tóm lại 1b/ cũng đã được loại trừ, và ta chỉ còn phải xét trường hợp “con”:

1c/ n  4. Lúc này, từ (1) và (2) dễ thấy deg G  deg H  2. Suy ra: R ( x)  2 và R ( x)  2 đều là các
đa thức bậc hai có hệ số bậc cao nhất bằng 2. Kết hợp với (5), ta thấy I  J  2. Lần này, với mọi
cặp chỉ số i, j mà R (ai )  R (a j ), dùng chính phương pháp chứng minh Nhận xét 4 ta có
2( ai  a j ) R ( ai )  R ( a j )  4 nên ( ai  a j ) | 2. Từ đây, vì a4  a1  2, ta suy ra R (a4 )  R (a1 ).

Nếu 1;4  J , thì R ( x)  2  2( x  a1 )( x  a4 ) và 2;3  I , nên

4  R (a2 )  2  2(a2  a1 )(a2  a4 )  0,

vô lý! Vậy, 1;4  I , nên R ( x)  2  2( x  a1 )( x  a4 ), 2;3  J và

4  R (a2 )  2  2(a2  a1 )(a2  a4 )  a2  a1  1, a4  a2  2;

4  R (a3 )  2  2( a3  a1 )( a3  a4 )  a3  a1  2, a4  a3  1.

Suy ra: a1 , a2 , a3 , a4 , theo thứ tự đó, là bốn số nguyên liên tiếp.

Thử lại: khi n  4, với a1 , a2 , a3 , a4 , theo thứ tự đó, là bốn số nguyên liên tiếp, ta có phân tích

S ( x )  ( x  a1 )( x  a1  1)( x  a1  2)( x  a1  3)  3
 [( x  a1 )( x  a1  3)  1][( x  a1 )( x  a1  3)  3]
thỏa mãn yêu cầu đề ra.

2. Trường hợp n  3.
3
Lúc này, deg G  2, deg H  1. Suy ra: H ( x)  x  a (a  ). Theo (1),  (a  a )  3.
i 1
i Nhưng

a  a1  a  a2  a  a3 nên chỉ có một khả năng: a  a1  1, a  a2  1, a  a3  3; tức là, các số
nguyên a1 , a2 , a3 lập thành một cấp số cộng với công sai 2.

Thử lại: khi n  3, với a1 , a2 , a3 là các số nguyên, theo thứ tự đó, lập thành một cấp số cộng với
công sai 2, ta có phân tích

S ( x )  ( x  a1 )( x  a1  2)( x  a1  4)  3
 ( x  a1  1)[( x  a1 ) 2  5( x  a1 )  3]

thỏa mãn yêu cầu đề ra.

3. Trường hợp n  2.
2
Lúc này, deg G  deg H  1. Suy ra: H ( x)  x  a, với a   nào đó. Theo (1),  (a  a )  3.
i 1
i

Nhưng a  a1  a  a2 nên chỉ có hai khả năng: a  a1  3, a  a2  1; hoặc a  a1  1, a  a2  3.


Vậy, a2  a1  2.

Thử lại: khi n  2, với a2  a1  2 ( a1   ), ta có phân tích

S ( x )  ( x  a1 )( x  a1  2)  3  ( x  a1  1)( x  a1  3)

thỏa mãn yêu cầu đề ra.

Kết luận: Tất cả các số nguyên n  2 và tất cả các bộ n số nguyên a1  a2    an thỏa mãn yêu
cầu đề ra gồm:

1. n  4; a1 , a2 , a3 , a4 , theo thứ tự đó, là bốn số nguyên liên tiếp;

2. n  3; các số nguyên a1 , a2 , a3 , theo thứ tự đó, lập thành một cấp số cộng với công sai 2;

3. n  2; a1 , a2 là các số nguyên mà a2  a1  2.

Bài toán 5 (Nga). Cho f ( x)  x 2  bx  c là một tam thức bậc hai với các hệ số b, c  . Giả sử
phương trình f  f ( x)   0 có đủ 4 nghiệm thực (không nhất thiết phân biệt), được ký hiệu bởi x1 ,
x2 , x3 , x4 . Biết x1  x2  1, chứng minh rằng c  1 4.
Lời giải. Nếu phương trình f ( x)  0 không có nghiệm thực thì: với mọi x  , đặt t : f ( x) ( ),
ta thấy f (t )  0; vậy, phương trình f  f ( x)   0 không có nghiệm thực, mâu thuẫn với giả thiết!
Do đó, phương trình f ( x)  0 phải có nghiệm thực. Giả sử {t1 ; t2 } là tập nghiệm của nó. Theo định
lý Viète,

t1  t2  b. (1)

Ngoài ra, f ( x)  ( x  t1 )( x  t2 ), nên

( x  x1 )( x  x2 )( x  x3 )( x  x4 )  f  f ( x)   ( f ( x)  t1 )( f ( x)  t2 )  ( x 2  bx  c  t1 )( x 2  bx  c  t2 ).

Từ đó, chỉ có 4 khả năng sau có thể xảy ra:

1. ( x  x1 ) | ( x 2  bx  c  t1 )  f ( x)  t1 và ( x  x2 ) | ( x 2  bx  c  t2 )  f ( x)  t2

2. ( x  x1 ) | ( x 2  bx  c  t2 )  f ( x)  t2 và ( x  x2 ) | ( x 2  bx  c  t1 )  f ( x)  t1

3. ( x  x1 )( x  x2 )  x 2  bx  c  t1 và ( x  x3 )( x  x4 )  x 2  bx  c  t2

4. ( x  x1 )( x  x2 )  x 2  bx  c  t2 và ( x  x3 )( x  x4 )  x 2  bx  c  t1.

Xét hai khả năng đầu. Lúc này

x12  x22  b( x1  x2 )  2c  f ( x1 )  f ( x2 )  t1  t2
(1)
 x12  x22  b  2c  b
x12  x22 x x 
2

c    1 2   1 4.
2  2 

Xét hai khả năng cuối. Lúc này (cân bằng hệ số bậc nhất hoặc dùng định lý Viète), ta có

1  x1  x2  b  b  1. (2)

Hơn nữa, phương trình f ( x)  ti  x 2  bx  c  ti  0 có biệt thức  i  b 2  4(c  ti )  0 với mỗi


i  {1; 2}. Vì vậy,

2b 2  8c  4(t1  t2 )  1   2  0
b 2  2(t1  t2 ) (1) b 2  2b (2)
c    1 4.
4 4

Bài toán 6 (Saudi Arabia TST 2016). Tìm số tự nhiên n  2 lớn nhất có tính chất: tồn tại n tam
thức bậc hai f1 ( x)  x 2  b1 x  c1 , f 2 ( x)  x 2  b2 x  c2 , , f n ( x)  x 2  bn x  cn đôi một không
trùng nhau, với các hệ số bi , ci   (1  i  n), sao cho, với mọi cặp chỉ số 1  i  j  n, phương
trình f i ( x)  f j ( x)  0 có và chỉ có một nghiệm thực.

Lời giải. Kiểm tra trực tiếp ta thấy 3 tam thức bậc hai f1 ( x) : x 2  4, f 2 ( x) : x 2  4 x  6,
f3 ( x) : x 2  8 x  12 thỏa hệ

 f1 ( x)  f 2 ( x)  2( x  1) 2

 f1 ( x)  f3 ( x)  2( x  2)
2

 f 2 ( x)  f3 ( x)  2( x  3) 2 .

Từ đó, n  3 có tính chất của đề bài.

Tiếp theo, ta chứng minh rằng n  3 là số tự nhiên lớn nhất cần tìm. Giả sử phản chứng rằng tồn tại
số tự nhiên n  4 có tính chất của đề bài. Ta sẽ đi đến mâu thuẫn bằng một trong các cách sau:

Cách 1: Từ giả thiết dễ dàng suy ra: với mọi cặp chỉ số 1  i  j  n, tồn tại số thực tij sao cho
f i ( x)  f j ( x)  2 x 2  (bi  b j ) x  ci  c j  2( x  tij ) 2 ; vì thế, bi  b j  4tij và ci  c j  2tij2 . Giải hệ

b1  b2  4t12

b1  b3  4t13
b2  b3  4t23

ta tìm ra

b1  2(t12  t13  t23 )



b2  2(t12  t13  t23 )
b3  2(t12  t13  t23 ).

Từ đó, 4t14  2(t12  t13  t23 )  b4  4t24  2(t12  t13  t23 ), và vì vậy,

t13  t24  t14  t23 . (1)

c1  c2  2t122 c1  t122  t132  t23


2

 
Tương tự, ta giải: c1  c3  2t132  c2  t122  t132  t23
2
và suy ra:
c2  c3  2t232 c3  t122  t132  t232
 

2t142  (t122  t132  t23


2
)  c4  2t24
2
 (t122  t132  t23
2
)  t142  t23
2
 t132  t24
2
. (2)

Từ (1) và (2), dễ thấy t13t24  t14t23 . Đặt S : t13  t24  t14  t23 và P : t13t24  t14t23 , ta có

{t13 ; t24 }  {t14 ; t23}


vì cả hai vế cùng là tập nghiệm của phương trình bậc hai x 2  Sx  P  0. Vậy, t13  t14 hoặc
t13  t23 . Nếu t13  t14 thì f1 ( x)  f3 ( x)  f1 ( x)  f 4 ( x)  f 3 ( x)  f 4 ( x), mâu thuẫn với giả thiết! Còn
nếu t13  t23 thì f1 ( x)  f 3 ( x)  f 2 ( x)  f 3 ( x)  f1 ( x)  f 2 ( x), cũng mâu thuẫn!

Cách 2: Với mọi cặp chỉ số 1  i  j  n, đặt g ij ( x) : f i ( x)  f j ( x) và gọi tij là nghiệm thực duy
4
nhất của phương trình g ij ( x)  0, ta có gij ( x)  2( x  tij ) 2 . Xét tổng g ( x) :  f i ( x), ta thấy:
i 1

2( x  t13 ) 2  2( x  t24 ) 2  g13 ( x)  g 24 ( x)  g ( x)  g14 ( x)  g 23 ( x)  2( x  t14 ) 2  2( x  t23 ) 2 .

Cân bằng hệ số hai vế ta cũng đi đến hệ (1)-(2) và tìm ra mâu thuẫn như đã thấy trong Cách 1.

Cách 3 (dựa trên ý tưởng của Nguyễn Thế Minh, giáo viên trường THPT chuyên Nguyễn Trãi, Hải
Dương): Theo giả thiết, với mọi cặp chỉ số 1  i  j  n, phương trình bậc hai

f i ( x)  f j ( x)  0  2 x 2  (bi  b j ) x  ci  c j  0

có nghiệm kép, nên có biệt thức  ij  0, suy ra (bi  b j ) 2  8(ci  c j ). Từ đó,

(b1  b2 ) 2  (b3  b4 ) 2  8(c1  c2  c3  c4 )  8(c1  c3  c2  c4 )  (b1  b3 ) 2  (b2  b4 ) 2


 (b1  b2 ) 2  (b1  b3 ) 2  (b2  b4 ) 2  (b3  b4 ) 2
 (b2  b3 )(2b1  b2  b3 )  (b2  b3 )(b2  b3  2b4 )
 (b2  b3 )(b1  b4 )  0.

Vậy, b2  b3 hoặc b1  b4 . Nếu b2  b3 , thì 8(c1  c2 )  (b1  b2 ) 2  (b1  b3 ) 2  8(c1  c3 )  c2  c3


nên f 2 ( x)  f 3 ( x), mâu thuẫn với giả thiết! Còn nếu b1  b4 , thì ta lại có
8(c1  c2 )  (b1  b2 ) 2  (b2  b4 ) 2  8(c2  c4 )  c1  c4 nên f1 ( x)  f 4 ( x), cũng mâu thuẫn!

Cách 4 (dựa trên ý tưởng của Nguyễn Văn Phương, giáo viên trường THPT chuyên Trần Đại Nghĩa,
TP. Hồ Chí Minh): Theo giả thiết, với mọi cặp chỉ số 1  i  j  n, phương trình bậc hai

f i ( x)  f j ( x)  0  2 x 2  (bi  b j ) x  ci  c j  0

có nghiệm kép, nên có biệt thức  ij  0, suy ra (bi  b j ) 2  8(ci  c j ). Từ đó, nếu n  3, ta thấy: với
mỗi chỉ số 3  j  n, bộ ( x; y )  (b j ; c j ) phải là một nghiệm của hệ phương trình

(b1  x) 2  8(c1  y )
 (3)
(b2  x)  8(c2  y ).
2
Nếu b1  b2 và hệ (3) có nghiệm thì c1  c2 ; suy ra f1 ( x)  f 2 ( x), mâu thuẫn với giả thiết! Nếu
b1  b2 , bằng cách trừ vế theo vế các phương trình trong hệ, ta thấy được hệ (3) có duy nhất nghiệm

 c1  c2 b1  b2
 x  4 b  b  2
 1 2

 y  (b1  x)  c .
2

 8
1

Vậy, n  3. Suy ra điều phải chứng minh.

Bài toán 7 (Saudi Arabia TST 2017). Đa thức P  [x] được gọi là có biểu diễn lập phương nếu nó
p
có dạng P( x)   i  Pi ( x)  , với p  *, (i )ip1  , ( Pi )ip1  [x] (phụ thuộc vào đa thức P).
3

i 1

Tính số tam thức bậc hai ax 2  bx  c có biểu diễn lập phương, với các hệ số a, b, c được lấy từ tập
S  {1; 2; 3; ; 2017} (tập gồm 2017 số nguyên dương đầu tiên).

n
Lời giải. Cho P( x)   ci xi , với n  , (ci )in0  . Ta nói đa thức P là đẹp nếu cả hai điều kiện
i 0

sau đây được thỏa mãn:

(i) với mọi chỉ số 0  i  n, nếu i  3 thì ci  3

(ii) 
i3
ci  6.
(0i  n )

Nhận xét 1: Mọi đa thức có biểu diễn lập phương thì đẹp.

Chứng minh: Trước hết, dễ thấy:

- nếu P1 và P2 là các đa thức đẹp thì đa thức P1  P2 cũng đẹp,

- nếu P là một đa thức đẹp và  là một số nguyên thì đa thức  P cũng đẹp.

Vì thế, từ định nghĩa của đa thức có biểu diễn lập phương, để suy ra Nhận xét 1 ta chỉ còn phải
chứng minh rằng đa thức P là đẹp nếu nó là lập phương của một đa thức với hệ số nguyên, tức là
nếu nó có dạng P( x) : Q( x)3 , trong đó, Q  [x]. (1)
m
Giả sử Q( x) :  ai xi , với m  , (ai )im0  , và P( x) : Q( x)3 . Áp dụng công thức “đa thức”:
i 0
N
 m  N!
 i    j0 ! j1 ! jm !
 0 j01 j1  m jm
 i 0  j0  j1  jm  N

(tổng ở vế phải được lấy theo tất cả các bộ số tự nhiên j0 , j1 , , jm có tổng bằng N , và lũy thừa
với số mũ 0 luôn được hiểu là bằng 1) với N : 3,  i : ai xi (0  i  m), ta thấy:

P( x)   ai3 x3i   3ai2 ak x 2i  k   6ai ak a x i  k   ; (2)


i ik ik
i 
k 

trong đó, với 3 tổng ở vế phải ta ngầm hiểu các chỉ số i, k ,  chạy từ 0 đến m, và đôi một khác
nhau. Hơn nữa, với mọi cặp chỉ số i, k như thế, ta có:

2i  k  3  2k  i  3;

và 3ai2 ak  3ak2 ai  3ai ak (ai  ak )  6. Vì thế, từ (2) ta thấy đa thức P là đẹp; khẳng định (1) đã được
chứng minh.

Nhận xét 2: Mọi đa thức đẹp với bậc bé hơn 3 đều có biểu diễn lập phương.

Chứng minh: Cho P là một đa thức đẹp với deg P  3. Khi đó, P( x)  3a2 x 2  3a1 x  a0 với a0 , a1 ,
a2 là các số nguyên mà a1  a2  2 ( a1  a2  2). Ta dễ dàng kiểm tra trực tiếp:

a1  a2 a a
P( x)  ( x  1)3  1 2 ( x  1)3  a1 x3  (a0  a2 )13 , (3)
2 2

suy ra: P có biểu diễn lập phương.

Trở lại bài toán: xét các tam thức bậc hai P ( x)  ax 2  bx  c, với các hệ số a, b, c được lấy từ tập
S . Theo các Nhận xét 1-2, P có biểu diễn lập phương khi và chỉ khi nó đẹp; tức là,

a  3, b  3, a  b  6. (4)

Tập T : {s  S | s  3}  {1  3; 2  3; 3  3; ; 672  3} gồm 672 số nguyên, trong đó có 336 số chẵn và


336 số lẻ. Điều kiện (4) tương đương với điều kiện: a  T , b  T , a và b có cùng tính chẵn lẻ.
Vậy, để P có biểu diễn lập phương, ta cần và chỉ cần:

- chọn a  T : có đúng 672 cách chọn

- chọn b  T mà b cùng tính chẵn lẻ với a : có cả thảy 336 cách chọn

- chọn c  S : có đúng 2017 cách chọn.


Theo quy tắc nhân, có cả thảy 672  336  2017  2  3362  2017 tam thức P có biểu diễn lập
phương.

Ghi chú: Ta có thể chứng minh mọi đa thức đẹp đều có biểu diễn lập phương. Nhận xét 2 trong lời
giải trên đây chính là bước cơ sở. Ở bước quy nạp, ta cho 2  n   và giả sử mọi đa thức đẹp với
bậc bé hơn 3(n  1) đều có biểu diễn lập phương. Xét P là một đa thức đẹp bất kỳ mà deg P  3n.
Ta cần chứng minh rằng P có biểu diễn lập phương. Dưới đây ta đưa ra hai cách chứng minh.

Cách 1: Theo giả thiết, ta có thể viết

P( x)  3a3n1 x3n1  3a3n2 x3n2  a3( n1) x 3( n1)  Q( x); (5)

trong đó, a3( n1) , a3n2 , a3n1 là các số nguyên; Q  [x], deg Q  3(n  1); và một trong hai khả
năng sau xảy ra:

(i) a3n2  a3n1 chẵn, đa thức Q đẹp

(ii) a3n2  a3n1 lẻ, đa thức Q( x)  3x3n 4 đẹp.

Xét (i). Lúc này, dựa trên biểu diễn (3) của các đa thức đẹp với bậc bé hơn 3, ta có thể viết lại (5)
dưới dạng:

P( x)  x3( n1)  3a3n1 x 2  3a3n2 x  a3( n1)   Q( x)


a a a a
 3n2 3n1 ( x n  x n1 )3  3n2 3n1 ( x n  x n1 )3  a3n2 ( x n )3  (a3( n1)  a3n1 )( x n1 )3  Q( x).
2 2

Từ đây, dùng giả thiết quy nạp ta suy ra P có biểu diễn lập phương.

Xét (ii). Lúc này, gợi ý một phần từ biểu diễn ở (i), ta viết lại (5) dưới dạng:

a3n2  1  a3n1 n a  1  a3n1 n


P( x)  ( x  x n1 )3  3n2 ( x  x n1 )3  a3n2 ( x n )3  ( x n  x n2 )3
2 2
 (a3( n1)  a3n1 )( x n1 )3  ( x n2 )3   Q( x)  3x3n4  .

Từ đây, dùng giả thiết quy nạp ta cũng suy ra P có biểu diễn lập phương.

Cách 2 (dựa trên ý tưởng của Nguyễn Duy Khang, giáo viên trường THPT chuyên Thoại Ngọc Hầu,
An Giang): Theo giả thiết, P là một đa thức đẹp với deg P  3n, nên ta có thể viết
n
P( x)   [3a3k 1 x3k 1  3a3k 2 x3k 2  a3( k 1) x3( k 1) ];
k 1
trong đó, (ai )i3n01  ,  3a  6, tức là  a  2.
i3
i
i3
i

Gọi d là phần dư trong phép chia a3n1  a3n2 cho 2, và đặt

Q( x) : P( x)  [3a3n1 x3n1  3(a3n2  d ) x 3n2  a3( n1) x 3( n1) ]  (3dx 3n2  3dx3( n1)1 ), (6)

ta thấy: Q là một đa thức đẹp có bậc bé hơn 3(n  1). Theo giả thiết quy nạp, Q có biểu diễn lập
phương. Định nghĩa (6) được viết lại thành

P( x)  ( x n1 )3[3a3n1 x 2  3(a3n2  d ) x  a3( n1) ]  d ( x n2 )3 (3x 4  3x 2 )  Q( x). (7)

Từ định nghĩa của d , ta thấy tam thức bậc hai 3a3n1 x 2  3(a3n 2  d ) x  a3( n1) đẹp, nên theo Nhận
xét 2 trong lời giải của Bài toán 7 (bước cơ sở) thì tam thức này có biểu diễn lập phương. Ngoài ra,
ta có thể kiểm tra trực tiếp “biểu diễn lập phương”: 3 x 4  3 x 2  ( x 2 )3  ( x 2  1)3  13. Cuối cùng, vì
tổng và tích của các đa thức có biểu diễn lập phương cũng là các đa thức có biểu diễn lập phương
(hãy chứng minh!) nên từ (7) suy ra P có biểu diễn lập phương.

Bài toán 8 (American Mathematical Monthly). Với mỗi số tự nhiên n và k (k  2), gọi ank là
thặng dư không âm bé nhất (mod k ) của Cn2 n ; tức là, ank    [0; k ) và Cn2 n  ank (mod k ). Tìm tất cả
các số tự nhiên k  2 để dãy số (ank )n0 có đuôi tuần hoàn; tức là,

n0  , T  *, n  n0 , ankT  ank

(nói cách khác, dãy số (ank )nn0 tuần hoàn khi n0 lớn; ta cũng nói dãy số (Cn2 n )n0 có đuôi tuần hoàn
(mod k )).

Lời giải 1 (của Nguyễn Cảnh Hoàng và Phạm Nam Khánh – thành viên đội tuyển Việt Nam dự thi
(2n)! (2n  1)!
IMO2017). Dễ thấy Cn2 n  2  2Cn2 n1  0(mod 2) n  1. Suy ra: k  2 là một số
n !n ! n !(n  1)!
tự nhiên thỏa yêu cầu đề bài.

Ta sẽ chứng minh rằng mọi số tự nhiên k  3 đều không thỏa yêu cầu đề bài. Thật vậy, vì k  3 nên
phải xảy ra một trong hai trường hợp sau:

(i) 4 | k

(ii) p | k với p là một số nguyên tố lẻ nào đó.

Ta sẽ dùng bổ đề sau đây (và một hệ quả hiển nhiên của nó):
Bổ đề: Với mọi x  , ta có [2 x]  2[x] khi (0 ){x}<1 2
2[x]  1 khi 1 2  {x} ( 1).

Chứng minh: dễ!

Hệ quả: x  , [2 x]  2[x].

Trở lại bài toán, trước tiên với trường hợp (i).

1. Với mỗi m  *, xét các số tự nhiên n thỏa 2m  n  2m1. Khi đó, 2m1  2n  2m 2 và

m 1
  2n   n 
2 (C2n n )     k   2  k  . (1)
k 1   2  2 

Tiếp theo, ta viết n  2t s với t : 2 (n)  , s  *, s lẻ, s  3 (vì n không phải là một lũy thừa
n n
đúng của 2). Do  s  3  2  ta thấy t  m. Theo Hệ quả ở trên, từ (1) ta có:
2t 2m

  2n   n     2n   n 
2 (C2n n )    t 1   2  t 1      m1   2  m1  
2  2  2  2 
(2)
  2n   n   2n   n 
   t 1   2  t 1    (1  2  0)   t 1   2  t 1   1.
2  2  2  2 

 n  s 1  2n   n 
Vì s lẻ nên  t 1      ; do đó, theo Bổ đề,  t 1   2  t 1   1. Vậy, (2) kéo theo
2  2 2 2  2 
2 (C2n n )  2, tức là Cn2 n  0(mod 4).

2. Nếu n  2m (m  *), ta lại có

m 1
  2n   n   m1 m
2 (C2n n )     k   2  k     2m1k  2 2mk  1,
k 1   2   2   k 1 k 1

nên Cn2 n  0(mod 4).

Vì khoảng cách giữa 2m và 2m1 có thể lớn tùy ý (khi m lớn), nên các kết quả ở hai bước 1-2 cho
thấy dãy số (Cn2 n )n0 không thể có đuôi tuần hoàn (mod 4), do đó nó cũng không thể có đuôi tuần
hoàn (mod k ).

Trường hợp (ii).


pm
1. Với mỗi m  *, xét các số tự nhiên n thỏa  n  p m . Khi đó, p m  2n  2 p m  p m1 nên
2
dùng Hệ quả ở trên ta có
m   2n   n    2n   n 
 p (C )     k   2  k     m   2  m   1  2  0  1;
n
2n
k 1   p   p   p  p 

tức là Cn2 n  0(mod p).

2. Nếu n  p m (m  *), ta lại có

m 
 2n   n  m
 
 p (Cn2 n )     k   2  k      2 p mk   2  p mk   0,
k 1   p   p   k 1

nên Cn2 n  0(mod p).

pm
Vì khoảng cách giữa và p m có thể lớn tùy ý (khi m lớn), nên các kết quả ở hai bước 1-2 cho
2
n 
thấy dãy số (C2 n ) n0 không thể có đuôi tuần hoàn (mod p), do đó nó cũng không thể có đuôi tuần
hoàn (mod k ).

Lời giải 2. Ta cũng nhận xét Cn2 n  2Cn2 n1  0(mod 2) n  1. Suy ra: k  2 là một số tự nhiên thỏa
yêu cầu đề bài. Tiếp theo, ta chứng minh rằng mọi số tự nhiên k  3 đều không thỏa yêu cầu đề bài,
vẫn theo cách xét một trong hai trường hợp sau:

(i) 4 | k

(ii) p | k với p là một số nguyên tố lẻ nào đó.

Trước hết, ta giới thiệu ký hiệu “đồng dư” trên các đa thức với hệ số nguyên: Cho d   * và
f , g  [x]. Nếu mọi hệ số của đa thức f  g đều là bội của d , ta sẽ viết f  d g ; và để cho tiện,
đôi khi ta cũng chấp nhận cách viết f ( x)  d g ( x). Nếu f ( x)  a   và g ( x)  b   là các đa thức
hằng thì “đồng dư thức” a  d b mới được giới thiệu quy về đồng dư thức a  b (mod d ) đã quen
biết trên các số nguyên. Quan hệ ''  d '' hiển nhiên là một quan hệ tương đương trên [x]. Ta cũng
có thể thực hiện các phép toán cộng/trừ/nhân vế theo vế trên các đồng dư thức vừa được giới thiệu,
với cùng “môđun” d (việc chứng minh là hoàn toàn dễ dàng).

Trở lại bài toán, đầu tiên là với trường hợp (i).
Với mỗi m  *, xét các số tự nhiên n có dạng n  2m  r với 0  r  2m. Khi đó, 2n  2m1  2r
nên

(1  x)   (1  x )  2 x  (1  x) 2 r
2m 1 2m
(1  x)  (1  x)
2n 2r 2

 
m
2
  (1  x 2 ) 2  2m (1  x 2 ) 2 1 (2 x)   Ci2m (1  x 2 ) 2 i (2 x)i  (1  x) 2 r
m m m

 i 2 
m 1
 4 (1  x ) (1  x)  4 (1  x ) (1  x)  4 
m 2
2 2 2r 2 2 2r

m 1
 4 (1  x 2 ) 2 (1  x) 2 r  (1  2 x 2  x 2 )(1  x) 2 r .
m m

So sánh hệ số của x n ở hai vế của đồng dư thức nói trên, ta suy ra

2  0 khi r  0
Cn2 n  4 2Cr2 r  4  r4
4C2 r 1  4 0 khi 1  r  2 .
m

Vì khoảng cách giữa 2m và 2m1 có thể lớn tùy ý (khi m lớn), nên kết quả trên cho thấy dãy số
(Cn2 n )n0 không thể có đuôi tuần hoàn (mod 4), do đó nó cũng không thể có đuôi tuần hoàn (mod k ).

Trường hợp (ii).

pm  r
Với mỗi m  *, xét các số tự nhiên n có dạng n  với 0  r  p m , r là số nguyên lẻ. Để ý
2
rằng 2n  p m  r và rằng Cip  p khi 1  i  p  1, ta có:

p m 1
 p 1

(1  x)  (1  x) (1  x)  1  x p   Cip xi  (1  x) r
2n pm r

 i 1 
p p m 1 p2 pm2
 p (1  x ) (1  x)  p (1  x ) (1  x) r  p 
r

   
 p (1  x p )(1  x) r    ai xi   x r  x p    ai xi 
m m

 ir   m 
 i p 

(ai là hệ số của xi trong đa thức (1  x p )(1  x) r ). So sánh hệ số của x n ở hai vế của đồng dư thức
m

nói trên, ta suy ra

1  1  2  p 0 khi r  p m ( r  n  p m )
C p 
n

khi 0  r  p m ( r  n  p m ).
2n
0
pm
Vì khoảng cách giữa và p m có thể lớn tùy ý (khi m lớn), nên kết quả trên cho thấy dãy số
2
(Cn2 n )n0 không thể có đuôi tuần hoàn (mod p), và do đó nó cũng không thể có đuôi tuần hoàn
(mod k ).

Vậy, k  2 là số tự nhiên duy nhất thỏa yêu cầu đề bài.

You might also like